Difference between revisions of "2004 AMC 12B Problems/Problem 24"
m (A more accurate diagram...) |
(→See also) |
||
(2 intermediate revisions by 2 users not shown) | |||
Line 20: | Line 20: | ||
\qquad\mathrm{(E)}\ 18</math> | \qquad\mathrm{(E)}\ 18</math> | ||
== Solution == | == Solution == | ||
− | {{ | + | Let <math>\alpha = DBC</math>. Then the first condition tells us that |
+ | <cmath> | ||
+ | \tan^2 DBE = \tan(DBE - \alpha)\tan(DBE + \alpha) = \frac {\tan^2 DBE - \tan^2 \alpha}{1 - \tan ^2 DBE \tan^2 \alpha}, | ||
+ | </cmath> | ||
+ | and multiplying out gives us <math>(\tan^4 DBE - 1) \tan^2 \alpha = 0</math>. Since <math>\tan\alpha \neq 0</math>, we have <math>\tan^4 DBE = 1 \Longrightarrow \angle DBE = 45^{\circ}</math>. | ||
+ | |||
+ | The second condition tells us that <math>2\cot (45 - \alpha) = 1 + \cot \alpha</math>. Expanding, we have <math>1 + \cot \alpha = 2\left[\frac {\cot \alpha + 1}{\cot \alpha - 1}\right] \Longrightarrow (\cot \alpha - 3)(\cot \alpha + 1) = 0</math>. Evidently <math>\cot \alpha \neq - 1</math>, so we get <math>\cot \alpha = 3</math>. | ||
+ | |||
+ | Now <math>BD = 5\sqrt {2}</math> and <math>AC = \frac {2BD} {\cot \alpha} = \frac {10\sqrt {2}}{3}</math>. Thus, <math>[ABC] = \frac {1}{2} \cdot 5\sqrt {2} \cdot \frac {10\sqrt {2}}{3} = \frac {50}{3}\ \mathrm{(B)}</math>. | ||
== See also == | == See also == | ||
Line 27: | Line 35: | ||
[[Category:Intermediate Geometry Problems]] | [[Category:Intermediate Geometry Problems]] | ||
[[Category:Intermediate Trigonometry Problems]] | [[Category:Intermediate Trigonometry Problems]] | ||
+ | {{MAA Notice}} |
Latest revision as of 18:59, 3 July 2013
Problem
In , , and is an altitude. Point is on the extension of such that . The values of , , and form a geometric progression, and the values of form an arithmetic progression. What is the area of ?
Solution
Let . Then the first condition tells us that and multiplying out gives us . Since , we have .
The second condition tells us that . Expanding, we have . Evidently , so we get .
Now and . Thus, .
See also
2004 AMC 12B (Problems • Answer Key • Resources) | |
Preceded by Problem 23 |
Followed by Problem 25 |
1 • 2 • 3 • 4 • 5 • 6 • 7 • 8 • 9 • 10 • 11 • 12 • 13 • 14 • 15 • 16 • 17 • 18 • 19 • 20 • 21 • 22 • 23 • 24 • 25 | |
All AMC 12 Problems and Solutions |
The problems on this page are copyrighted by the Mathematical Association of America's American Mathematics Competitions.